Difference between revisions of "2018 AMC 10A Problems/Problem 1"

m (Solution)
m (minor edit)
(11 intermediate revisions by 9 users not shown)
Line 1: Line 1:
 
== Problem ==
 
== Problem ==
 
What is the value of
 
What is the value of
<cmath>\left(\left((2+1)^{-1}+1\right)^{-1}+1\right)^{-1}+1?</cmath><math>\textbf{(A) } \frac58 \qquad \textbf{(B) }\frac{11}7 \qquad \textbf{(C) } \frac85 \qquad \textbf{(D) } \frac{18}{11} \qquad \textbf{(E) } \frac{15}8 </math>
+
<cmath>\left(\left((2+1)^{-1}+1\right)^{-1}+1\right)^{-1}+1?</cmath>
 +
 
 +
<math>\textbf{(A) } \frac58 \qquad \textbf{(B) }\frac{11}7 \qquad \textbf{(C) } \frac85 \qquad \textbf{(D) } \frac{18}{11} \qquad \textbf{(E) } \frac{15}8 </math>
  
 
== Solution ==  
 
== Solution ==  
<math>2+1=3</math>, so the reciprocal is <math>\frac{1}{3}</math>. Adding <math>1</math> and repeating, leads to <math>\frac{3}{4}</math>. Rinse and repeat to get <math>\frac{4}{7}</math>. Adding <math>1</math> to that is <math>\frac{11}{7} </math> <math>\boxed{\textbf{(B)}}</math>
+
<cmath> \left(\left((2+1)^{-1}+1\right)^{-1}+1\right)^{-1}+1 </cmath>
 +
<cmath> =\left(\left(3)^{-1}+1\right)^{-1}+1\right)^{-1}+1 </cmath>
 +
<cmath> =\left(\left(\frac{1}{3}+1\right)^{-1}+1\right)^{-1}+1 </cmath>
 +
<cmath> =\left(\left(\frac{4}{3}\right)^{-1}+1\right)^{-1}+1 </cmath>
 +
<cmath> =\left(\frac{3}{4}+1\right)^{-1}+1 </cmath>
 +
<cmath> =\left(\frac{7}{4}\right)^{-1}+1 </cmath>
 +
<cmath> =\frac{4}{7}+1 </cmath>
 +
<cmath> =\frac{11}{7} </cmath>
 +
Therefore, the answer is <math>\boxed{\textbf{(B) } \frac{11}{7} }</math>.
 +
 
 +
== Video Solutions ==
 +
https://youtu.be/vO-ELYmgRI8
 +
 
 +
https://youtu.be/cat3yTIpX4k
 +
 
 +
~savannahsolver
  
 
== See Also ==
 
== See Also ==
 
 
{{AMC10 box|year=2018|ab=A|before=First Problem|num-a=2}}
 
{{AMC10 box|year=2018|ab=A|before=First Problem|num-a=2}}
 
{{MAA Notice}}
 
{{MAA Notice}}

Revision as of 01:45, 19 October 2020

Problem

What is the value of \[\left(\left((2+1)^{-1}+1\right)^{-1}+1\right)^{-1}+1?\]

$\textbf{(A) } \frac58 \qquad \textbf{(B) }\frac{11}7 \qquad \textbf{(C) } \frac85 \qquad \textbf{(D) } \frac{18}{11} \qquad \textbf{(E) } \frac{15}8$

Solution

\[\left(\left((2+1)^{-1}+1\right)^{-1}+1\right)^{-1}+1\] \[=\left(\left(3)^{-1}+1\right)^{-1}+1\right)^{-1}+1\] \[=\left(\left(\frac{1}{3}+1\right)^{-1}+1\right)^{-1}+1\] \[=\left(\left(\frac{4}{3}\right)^{-1}+1\right)^{-1}+1\] \[=\left(\frac{3}{4}+1\right)^{-1}+1\] \[=\left(\frac{7}{4}\right)^{-1}+1\] \[=\frac{4}{7}+1\] \[=\frac{11}{7}\] Therefore, the answer is $\boxed{\textbf{(B) } \frac{11}{7} }$.

Video Solutions

https://youtu.be/vO-ELYmgRI8

https://youtu.be/cat3yTIpX4k

~savannahsolver

See Also

2018 AMC 10A (ProblemsAnswer KeyResources)
Preceded by
First Problem
Followed by
Problem 2
1 2 3 4 5 6 7 8 9 10 11 12 13 14 15 16 17 18 19 20 21 22 23 24 25
All AMC 10 Problems and Solutions

The problems on this page are copyrighted by the Mathematical Association of America's American Mathematics Competitions. AMC logo.png